Difference between revisions of "1997 AHSME Problems/Problem 21"

(Created page with "== See also == {{AHSME box|year=1997|num-b=20|num-a=22}}")
 
Line 1: Line 1:
 +
==Problem 21==
 +
 +
For any positive integer <math>n</math>, let
 +
 +
<math> f(n) =\left\{\begin{matrix}\log_{8}{n}, &\text{if }\log_{8}{n}\text{ is rational,}\\ 0, &\text{otherwise.}\end{matrix}\right. </math>
 +
 +
What is <math> \sum_{n = 1}^{1997}{f(n)} </math>?
 +
 +
<math> \textbf{(A)}\ \log_{8}{2047}\qquad\textbf{(B)}\ 6\qquad\textbf{(C)}\ \frac{55}{3}\qquad\textbf{(D)}\ \frac{58}{3}\qquad\textbf{(E)}\ 585 </math>
 +
 +
 
== See also ==
 
== See also ==
 
{{AHSME box|year=1997|num-b=20|num-a=22}}
 
{{AHSME box|year=1997|num-b=20|num-a=22}}

Revision as of 18:12, 9 August 2011

Problem 21

For any positive integer $n$, let

$f(n) =\left\{\begin{matrix}\log_{8}{n}, &\text{if }\log_{8}{n}\text{ is rational,}\\ 0, &\text{otherwise.}\end{matrix}\right.$

What is $\sum_{n = 1}^{1997}{f(n)}$?

$\textbf{(A)}\ \log_{8}{2047}\qquad\textbf{(B)}\ 6\qquad\textbf{(C)}\ \frac{55}{3}\qquad\textbf{(D)}\ \frac{58}{3}\qquad\textbf{(E)}\ 585$


See also

1997 AHSME (ProblemsAnswer KeyResources)
Preceded by
Problem 20
Followed by
Problem 22
1 2 3 4 5 6 7 8 9 10 11 12 13 14 15 16 17 18 19 20 21 22 23 24 25 26 27 28 29 30
All AHSME Problems and Solutions